Difference between revisions of "2021 AMC 10A Problems/Problem 3"

m (Problem)
Line 1: Line 1:
==Problem==
 
If a circle is inscribed in a square and then have right triangles with legs on the sides of the square and within the area between the circle and the square, what is the area inside the square but outside the triangles and the circle if the area of the circle is equal to the perimeter of the square?
 
 
<math> \textbf{(A)}\ 12pi + 14\qquad\textbf{(B)}\ 11pi\qquad\textbf{(C)}\ 10\qquad\textbf{(D)}\ 32pi\qquad\textbf{(E)}\ 22pi </math>
 
 
==Solution==
 
We don't know yet, but since 10+9=21, we know that the answer is G.
 
 
==Note==
 
This problem might also be on the AMC 12A. If so, please redirect it there.
 
 
==See also==
 
==See also==
 
{{AMC10 box|year=2021|ab=A|num-b=2|num-a=4}}
 
{{AMC10 box|year=2021|ab=A|num-b=2|num-a=4}}
 
{{MAA Notice}}
 
{{MAA Notice}}

Revision as of 21:35, 31 January 2021

See also

2021 AMC 10A (ProblemsAnswer KeyResources)
Preceded by
Problem 2
Followed by
Problem 4
1 2 3 4 5 6 7 8 9 10 11 12 13 14 15 16 17 18 19 20 21 22 23 24 25
All AMC 10 Problems and Solutions

The problems on this page are copyrighted by the Mathematical Association of America's American Mathematics Competitions. AMC logo.png